K
Khách

Hãy nhập câu hỏi của bạn vào đây, nếu là tài khoản VIP, bạn sẽ được ưu tiên trả lời.

16 tháng 12 2019

S>2

 nhân s với 2

lấy 2 S - S = 1+ 1/2 + 1/22015

:)) HD thui

16 tháng 12 2019

Ta có: \(S=1+\frac{1}{2}+\frac{1}{2^2}+\frac{1}{2^3}...+\frac{1}{2^{2014}}\)

\(2S=2+1+\frac{1}{2}+\frac{1}{2^2}+...+\frac{1}{2^{2013}}\)

\(2S-S=2-\frac{1}{2^{2014}}\)

Hay \(S=2-\frac{1}{2^{2014}}< 2\)

Suy ra: \(S< 2\)

_Học tốt_

18 tháng 12 2016

lớn hơn , bé hơn hoặc bằng dễ òm đi chịch hk cưng ?

18 tháng 12 2016

ĐANG CẦN GẤP

25 tháng 2 2017

\(S=\frac{1}{\sqrt{1}}+\frac{1}{\sqrt{2}-\sqrt{1}}+\frac{1}{\sqrt{3}-\sqrt{2}}+...+\frac{1}{\sqrt{2025}-\sqrt{2024}}\)

Ta nhận xét thấy mỗi số hạng trong S đều dương. Từ đó ta đặt

\(A=\frac{1}{\sqrt{1}}+\frac{1}{\sqrt{2}-\sqrt{1}}+\frac{1}{\sqrt{3}-\sqrt{2}}+...+\frac{1}{\sqrt{2024}-\sqrt{2023}}\left(A>0\right)\)

\(\Rightarrow S=A+\frac{1}{\sqrt{2025}-\sqrt{2024}}=A+\frac{\sqrt{2025}+\sqrt{2024}}{\left(\sqrt{2025}-\sqrt{2024}\right)\left(\sqrt{2025}+\sqrt{2024}\right)}\)

\(=A+\sqrt{2025}+\sqrt{2024}>\sqrt{2025}=45\)

Vậy \(S>45\)

PS: Phan Thanh Tịnh xem lại bài giải nhé bạn

24 tháng 2 2017

Ta có : 1 = (n + 1) - n =\(\left(\sqrt{n+1}\right)^2-\left(\sqrt{n}\right)^2\)

\(=\left(\sqrt{n+1}\right)^2-\sqrt{n+1}.\sqrt{n}+\sqrt{n+1}.\sqrt{n}+\left(\sqrt{n}\right)^2\)

\(=\sqrt{n+1}.\left(\sqrt{n+1}-\sqrt{n}\right)+\sqrt{n}.\left(\sqrt{n+1}-\sqrt{n}\right)\)

\(=\left(\sqrt{n+1}-\sqrt{n}\right)\left(\sqrt{n-1}+\sqrt{n}\right)\)\

\(\Rightarrow\frac{1}{\sqrt{n+1}-\sqrt{n}}=\sqrt{n+1}+\sqrt{n}\)

Áp dụng vào bài toán,ta có :

\(S=\sqrt{1}+\sqrt{2}-\sqrt{1}+\sqrt{3}-\sqrt{2}+...+\sqrt{2025}-\sqrt{2024}=\sqrt{2025}\)= 45

Vậy S = 45

9 tháng 12 2019

Ta có: \(\frac{1}{2^2}=\frac{1}{2.2}< \frac{1}{1.2}\)

Tương tự : \(\frac{1}{3^2}< \frac{1}{2.3}\)\(\frac{1}{4^2}< \frac{1}{3.4}\); ......... ; \(\frac{1}{2014^2}< \frac{1}{2013.2014}\)

\(\Rightarrow S< \frac{1}{1.2}+\frac{1}{2.3}+\frac{1}{3.4}+........+\frac{1}{2013.2014}\)               

        \(=1-\frac{1}{2}+\frac{1}{2}-\frac{1}{3}+\frac{1}{3}-\frac{1}{4}+.........+\frac{1}{2013}-\frac{1}{2014}\)

        \(=1-\frac{1}{2014}=\frac{2013}{2014}\)

\(\Rightarrow S< \frac{2013}{2014}\left(đpcm\right)\)

DD
27 tháng 5 2021

\(S=2014+\frac{2014}{1+2}+\frac{2014}{1+2+3}+...+\frac{2014}{1+2+3+...+10000}\)

\(S=\frac{2014}{\frac{1.2}{2}}+\frac{2014}{\frac{2.3}{2}}+\frac{2014}{\frac{3.4}{2}}+...+\frac{2014}{\frac{10000.10001}{2}}\)

\(S=\frac{4028}{1.2}+\frac{4028}{2.3}+\frac{4028}{3.4}+...+\frac{4028}{10000.10001}\)

\(S=4028\left(\frac{1}{1.2}+\frac{1}{2.3}+\frac{1}{3.4}+...+\frac{1}{10000.10001}\right)\)

\(S=4028\left(\frac{2-1}{1.2}+\frac{3-2}{2.3}+\frac{4-3}{3.4}+...+\frac{10001-10000}{10000.10001}\right)\)

\(S=4028\left(1-\frac{1}{2}+\frac{1}{2}-\frac{1}{3}+...+\frac{1}{10000}-\frac{1}{10001}\right)\)

\(S=4028\left(1-\frac{1}{10001}\right)=\frac{40280000}{10001}\)

14 tháng 8 2016

\(S=\frac{3}{1^2\cdot2^2}+\frac{5}{2^2\cdot3^2}+\frac{7}{3^2\cdot4^2}+...+\frac{99}{49^2\cdot50^2}\)

\(=\frac{1}{1^2}-\frac{1}{2^2}+\frac{1}{2^2}-\frac{1}{3^2}+.....+\frac{1}{49^2}-\frac{1}{50^2}\)

\(=1-\frac{1}{50^2}=\frac{2499}{2500}\)

\(T=\frac{1}{\left(2-1\right)\left(2+1\right)}+\frac{1}{\left(3-1\right)\left(3+1\right)}+...+\frac{1}{\left(50-1\right)\left(50+1\right)}\)

\(=\frac{1}{1\cdot3}+\frac{1}{2\cdot4}+\frac{1}{3\cdot5}+...+\frac{1}{49\cdot51}\)

\(=\frac{1}{2}\cdot\left(\frac{1}{1}-\frac{1}{3}+\frac{1}{2}-\frac{1}{4}+\frac{1}{3}-\frac{1}{5}+...+\frac{1}{49}-\frac{1}{51}\right)\)

\(=\frac{1}{2}\cdot\left(1+\frac{1}{2}-\frac{1}{51}\right)=\frac{151}{204}\)

Vì \(\frac{2499}{2500}>\frac{151}{204}\)nên S>T

14 tháng 8 2016

JOKER_Võ Văn Quốc, T = \(\frac{1}{2}.\left(1-\frac{1}{51}+\frac{1}{2}-\frac{1}{50}\right)\)mới đúng
Sẽ dễ hơn nếu bạn chia ra 2 vế \(\frac{1}{3.5}+\frac{1}{5.7}+...+\frac{1}{49.51}\)và \(\frac{1}{2.4}+\frac{1}{4.6}+...+\frac{1}{48+50}\)

7 tháng 8 2015

\(4.M=4.\left(\frac{1}{4}+\frac{2}{4^2}+\frac{3}{4^3}+...+\frac{2014}{4^{2014}}\right)=1+\frac{2}{4}+\frac{3}{4^2}+...+\frac{2014}{4^{2013}}\)

=> 4M - M = \(1+\left(\frac{2}{4}-\frac{1}{4}\right)+\left(\frac{3}{4^2}-\frac{2}{4^2}\right)+...+\left(\frac{2014}{4^{2013}}-\frac{2013}{4^{2013}}\right)-\frac{2014}{4^{2014}}\)

=> 3.M = \(1+\frac{1}{4}+\frac{1}{4^2}+...+\frac{1}{4^{2013}}-\frac{2014}{4^{2014}}\)

Tính \(N=1+\frac{1}{4}+\frac{1}{4^2}+...+\frac{1}{4^{2013}}\)

=> \(4.N=4+1+\frac{1}{4}+\frac{1}{4^2}+...+\frac{1}{4^{2012}}\)

=> 4N - N = 4 - \(\frac{1}{4^{2013}}\)=> N = \(\frac{4}{3}-\frac{1}{3.4^{2013}}\)=> N < 4/3

Ta có:  3M < N => M < N/3 => M < (4/3)/3 = 2/9

vậy M < 4/9

 

8 tháng 3 2018

bạn tham khảo tạm ở đây nhé

https://olm.vn/hoi-dap/question/994432.html

^^

8 tháng 3 2018

bạn tham khảo tại đây nhé

http://olm.vn/hoi-dap/question/994432.html

^-^

4 tháng 4 2016

Bạn xem lại đề câu a) cho rõ lại

Câu b) Tại x=2013 thì B=x2013-(x+1)x2012+(x+1)x2011-(x+1)x2010+...-(x+1)x2+(x+1)x-1

                                 = x2013-x2013-x2012+x2012+x2011-x2011-x2010+..-x3 - x2+x2+x-1

                                 = x-1 =  2012

27 tháng 3 2017

phải là so sánh A với 2 mới đúng